(Wrote expansion of electrical energy)
(Added headings for an exhaustive W_E calculation)
Line 14: Line 14:
 
----
 
----
 
==Question 2==
 
==Question 2==
 +
===<small>Setting Up the Problem</small>===
 +
 
The following information about the '''circuit model''' of the transformer may be obtained from the figure:
 
The following information about the '''circuit model''' of the transformer may be obtained from the figure:
 
* Winding resistances: <math>r_1 = r_2 = 1 \, \Omega</math>
 
* Winding resistances: <math>r_1 = r_2 = 1 \, \Omega</math>
Line 51: Line 53:
 
\end{align}</math>
 
\end{align}</math>
  
The electrical energy supplied to this transformer may be found by combining the voltage equations and flux linkage equations to the first equation for <math>W_E</math>. The leakage terms are omitted since <math>L_{\ell 1} = L_{\ell 2} = 0</math>. <math>\tau</math> is used as a dummy variable for indefinite integrals.
+
===<small>Method 1: Exhaustive Calculation</small>===
 +
 
 +
The electrical energy supplied to this transformer after a long time may be found by combining the voltage equations and flux linkage equations to the first equation for <math>W_E</math>. The leakage terms are omitted since <math>L_{\ell 1} = L_{\ell 2} = 0</math>. <math>\tau</math> is used as a dummy variable for integrals.
  
 
<math>\begin{align}
 
<math>\begin{align}
Line 58: Line 62:
 
&{}+ \int_{\tau = 0}^{1 \, \textrm{s}} r_2 \cancelto{0}{i_2^2} \, d\tau + \int_{\tau = 1 \, \textrm{s}}^{t} r_2 i_2^2 \, d\tau \\
 
&{}+ \int_{\tau = 0}^{1 \, \textrm{s}} r_2 \cancelto{0}{i_2^2} \, d\tau + \int_{\tau = 1 \, \textrm{s}}^{t} r_2 i_2^2 \, d\tau \\
 
&{}+ \int_{\tau = 0}^{1 \, \textrm{s}} L_{m2} \cancelto{0}{i_2} \frac{di_2}{\cancel{d\tau}} \, \cancel{d\tau} + \int_{\tau = 1 \, \textrm{s}}^{t} L_{m2} i_2 \frac{di_2}{\cancel{d\tau}} \, \cancel{d\tau} + \int_{\tau = 0}^{1 \, \textrm{s}} \frac{N_1}{N_2} L_{m2} \cancelto{0}{i_2} \frac{di_1}{\cancel{d\tau}} \, \cancel{d\tau} + \int_{\tau = 1 \, \textrm{s}}^{t} \frac{N_1}{N_2} L_{m2} i_2 \frac{di_1}{\cancel{d\tau}} \, \cancel{d\tau} \\
 
&{}+ \int_{\tau = 0}^{1 \, \textrm{s}} L_{m2} \cancelto{0}{i_2} \frac{di_2}{\cancel{d\tau}} \, \cancel{d\tau} + \int_{\tau = 1 \, \textrm{s}}^{t} L_{m2} i_2 \frac{di_2}{\cancel{d\tau}} \, \cancel{d\tau} + \int_{\tau = 0}^{1 \, \textrm{s}} \frac{N_1}{N_2} L_{m2} \cancelto{0}{i_2} \frac{di_1}{\cancel{d\tau}} \, \cancel{d\tau} + \int_{\tau = 1 \, \textrm{s}}^{t} \frac{N_1}{N_2} L_{m2} i_2 \frac{di_1}{\cancel{d\tau}} \, \cancel{d\tau} \\
 +
\end{align}</math>
 +
 +
All the zero terms can be removed, and time integrals with inductance are replaced with current integrals using the currents at the time bounds.
 +
 +
<math>\begin{align}
 +
W_E &= \int_{\tau = 0}^{1 \, \textrm{s}} r_1 i_1^2 \, d\tau + \int_{i_1 = 10 \, \textrm{A}}^{6.32 \, \textrm{A}} L_{m1} i_1 \, di_1 + \int_{i_2 = 0}^{0} \cancelto{1}{\frac{N_2}{N_1}} L_{m1} i_1 \, di_2 \\
 +
&{} + \int_{\tau = 1 \, \textrm{s}}^{t} r_2 i_2^2 \, d\tau + \int_{i_2 = 0}^{i_2(t), \, t \geq 1 \, \textrm{s}} L_{m2} i_2 \, di_2 + \int_{i_1 = 0}^{0} \cancelto{1}{\frac{N_1}{N_2}} L_{m2} i_2 \, di_1
 +
\end{align}</math>
 +
 +
Any integration between identical bounds will have a zero result. Because the secondary winding is short circuited <math>v_2(t) = 0, \; t \geq 1 \, \textrm{s}</math>, it is further known that <math>\int_{\tau = 1 \, \textrm{s}}^{t} r_2 i_2^2 \, d\tau + \int_{i_2 = 0}^{i_2(t), \, t \geq 1 \, \textrm{s}} L_{m2} i_2 \, di_2 = 0</math> as a consequence of the voltage equations. The electrical energy waveform must account for the period <math>0 \leq t < 1 \, \textrm{s}</math> and revert to time integrals using <math>\frac{di_1}{dt} = +10 e^{-t} \, \frac{\textrm{A}}{\textrm{s}}</math>.
 +
 +
<math>\begin{align}
 +
W_E(t) &=
 +
\begin{cases}
 +
\int_{\tau = 0}^{t} r_1 i_1^2 \, d\tau + \int_{\tau = 0}^{t} L_{m1} i_1 \frac{di_1}{d\tau} \, d\tau, \; &0 \leq t < 1 \, \textrm{s} \\
 +
\int_{\tau = 0}^{1 \, \textrm{s}} r_1 i_1^2 \, d\tau + \int_{\tau = 0}^{1 \, \textrm{s}} L_{m1} i_1 \frac{di_1}{d\tau} \, d\tau, \; &t \geq 1 \, \textrm{s}
 +
\end{cases} \\
 +
 +
W_E(t) &=
 +
\begin{cases}
 +
\int_{\tau = 0}^{t} 100 \, \textrm{W} \left(1 - e^{-t}\right)^2 \, d\tau + \int_{\tau = 0}^{t} 100 \, \textrm{W} \left(1 - e^{-t}\right) e^{-t} \, d\tau, \; &0 \leq t < 1 \, \textrm{s} \\
 +
\int_{\tau = 0}^{1 \, \textrm{s}} 100 \, \textrm{W} \left(1 - e^{-t}\right)^2 \, d\tau + \int_{\tau = 0}^{1 \, \textrm{s}} 100 \, \textrm{W} \left(1 - e^{-t}\right) e^{-t} \, d\tau, \; &t \geq 1 \, \textrm{s}
 +
\end{cases}
 
\end{align}</math>
 
\end{align}</math>
  

Revision as of 18:26, 10 January 2018


Answers and Discussions for

ECE Ph.D. Qualifying Exam ES-1 August 2007



Question 2

Setting Up the Problem

The following information about the circuit model of the transformer may be obtained from the figure:

  • Winding resistances: $ r_1 = r_2 = 1 \, \Omega $
  • Winding leakage inductances: $ L_{\ell 1} = L_{\ell 2} = 0 $
  • Magnetizing inductances: $ L_{m1} = L_{m2} = 1 \, \textrm{H} $
  • Winding turns: $ N_1 = N_2 = 10 $

The following information about the circuit excitation and state of the transformer is given in the text (note that it is easier to avoid referred quantities because of how the information is given):

  • Initial primary winding voltage: $ v_1(0) = 10 \, \textrm{V} $
  • Secondary winding current (initially open circuited): $ i_2(t) = 0, \; 0 \leq t < 1 \, \textrm{s} $
  • Primary winding current (becomes open circuited): $ i_1(t) = \begin{cases} 10 \left(1 - e^{-t}\right) \, \textrm{A} \; &0 \leq t < 1 \, \textrm{s} \\ 0 \; &t \geq 1 \, \textrm{s} \end{cases} $
    • For convenience, it is given that $ i_1 \left(t = 1^- \, \textrm{s}\right) = 6.32 \, \textrm{A} $
  • Secondary winding voltage (becomes short circuited): $ v_2(t) = 0, \; t \geq 1 \, \textrm{s} $

The electrical energy provided to the machine is denoted $ W_E $.

$ \begin{equation} W_E = \sum_{k = 1}^{N} \int v_k i_k \, dt \end{equation} $

From KVL, the voltage equations needed for the energy calculation are found ($ p $ denotes the Heaviside operator).

$ \begin{align} v_1 &= r_1 i_1 + p\lambda_1 \\ v_2 &= r_2 i_2 + p\lambda_2 \end{align} $

The flux linkage equations for a transformer are used. Recall that the magnetic flux linked to winding 1 due to a current in winding 2 is $ \Phi_{1,2} = L_{m1} \frac{N_2}{N_1} i_2 $.


$ \begin{align} \lambda_1 &= L_{\ell 1} i_1 + L_{m1} \left(i_1 + \frac{N_2}{N_1} i_2\right) \\ \lambda_2 &= L_{\ell 2} i_2 + L_{m2} \left(i_2 + \frac{N_1}{N_2} i_1\right) \end{align} $

Method 1: Exhaustive Calculation

The electrical energy supplied to this transformer after a long time may be found by combining the voltage equations and flux linkage equations to the first equation for $ W_E $. The leakage terms are omitted since $ L_{\ell 1} = L_{\ell 2} = 0 $. $ \tau $ is used as a dummy variable for integrals.

$ \begin{align} W_E &= \int_{\tau = 0}^{1 \, \textrm{s}} r_1 i_1^2 \, d\tau + \int_{\tau = 1 \, \textrm{s}}^{t} r_1 \cancelto{0}{i_1^2} \, d\tau \\ &{}+ \int_{\tau = 0}^{1 \, \textrm{s}} L_{m1} i_1 \frac{di_1}{\cancel{d\tau}} \, \cancel{d\tau} + \int_{\tau = 1 \, \textrm{s}}^{t} L_{m1} \cancelto{0}{i_1} \frac{di_1}{\cancel{d\tau}} \, \cancel{d\tau} + \int_{\tau = 0}^{1 \, \textrm{s}} \frac{N_2}{N_1} L_{m1} i_1 \frac{di_2}{\cancel{d\tau}} \, \cancel{d\tau} + \int_{\tau = 1 \, \textrm{s}}^{t} \frac{N_2}{N_1} L_{m1} \cancelto{0}{i_1} \frac{di_2}{\cancel{d\tau}} \, \cancel{d\tau} \\ &{}+ \int_{\tau = 0}^{1 \, \textrm{s}} r_2 \cancelto{0}{i_2^2} \, d\tau + \int_{\tau = 1 \, \textrm{s}}^{t} r_2 i_2^2 \, d\tau \\ &{}+ \int_{\tau = 0}^{1 \, \textrm{s}} L_{m2} \cancelto{0}{i_2} \frac{di_2}{\cancel{d\tau}} \, \cancel{d\tau} + \int_{\tau = 1 \, \textrm{s}}^{t} L_{m2} i_2 \frac{di_2}{\cancel{d\tau}} \, \cancel{d\tau} + \int_{\tau = 0}^{1 \, \textrm{s}} \frac{N_1}{N_2} L_{m2} \cancelto{0}{i_2} \frac{di_1}{\cancel{d\tau}} \, \cancel{d\tau} + \int_{\tau = 1 \, \textrm{s}}^{t} \frac{N_1}{N_2} L_{m2} i_2 \frac{di_1}{\cancel{d\tau}} \, \cancel{d\tau} \\ \end{align} $

All the zero terms can be removed, and time integrals with inductance are replaced with current integrals using the currents at the time bounds.

$ \begin{align} W_E &= \int_{\tau = 0}^{1 \, \textrm{s}} r_1 i_1^2 \, d\tau + \int_{i_1 = 10 \, \textrm{A}}^{6.32 \, \textrm{A}} L_{m1} i_1 \, di_1 + \int_{i_2 = 0}^{0} \cancelto{1}{\frac{N_2}{N_1}} L_{m1} i_1 \, di_2 \\ &{} + \int_{\tau = 1 \, \textrm{s}}^{t} r_2 i_2^2 \, d\tau + \int_{i_2 = 0}^{i_2(t), \, t \geq 1 \, \textrm{s}} L_{m2} i_2 \, di_2 + \int_{i_1 = 0}^{0} \cancelto{1}{\frac{N_1}{N_2}} L_{m2} i_2 \, di_1 \end{align} $

Any integration between identical bounds will have a zero result. Because the secondary winding is short circuited $ v_2(t) = 0, \; t \geq 1 \, \textrm{s} $, it is further known that $ \int_{\tau = 1 \, \textrm{s}}^{t} r_2 i_2^2 \, d\tau + \int_{i_2 = 0}^{i_2(t), \, t \geq 1 \, \textrm{s}} L_{m2} i_2 \, di_2 = 0 $ as a consequence of the voltage equations. The electrical energy waveform must account for the period $ 0 \leq t < 1 \, \textrm{s} $ and revert to time integrals using $ \frac{di_1}{dt} = +10 e^{-t} \, \frac{\textrm{A}}{\textrm{s}} $.

$ \begin{align} W_E(t) &= \begin{cases} \int_{\tau = 0}^{t} r_1 i_1^2 \, d\tau + \int_{\tau = 0}^{t} L_{m1} i_1 \frac{di_1}{d\tau} \, d\tau, \; &0 \leq t < 1 \, \textrm{s} \\ \int_{\tau = 0}^{1 \, \textrm{s}} r_1 i_1^2 \, d\tau + \int_{\tau = 0}^{1 \, \textrm{s}} L_{m1} i_1 \frac{di_1}{d\tau} \, d\tau, \; &t \geq 1 \, \textrm{s} \end{cases} \\ W_E(t) &= \begin{cases} \int_{\tau = 0}^{t} 100 \, \textrm{W} \left(1 - e^{-t}\right)^2 \, d\tau + \int_{\tau = 0}^{t} 100 \, \textrm{W} \left(1 - e^{-t}\right) e^{-t} \, d\tau, \; &0 \leq t < 1 \, \textrm{s} \\ \int_{\tau = 0}^{1 \, \textrm{s}} 100 \, \textrm{W} \left(1 - e^{-t}\right)^2 \, d\tau + \int_{\tau = 0}^{1 \, \textrm{s}} 100 \, \textrm{W} \left(1 - e^{-t}\right) e^{-t} \, d\tau, \; &t \geq 1 \, \textrm{s} \end{cases} \end{align} $


Discussion



Back to ES-1, August 2007

Alumni Liaison

Meet a recent graduate heading to Sweden for a Postdoctorate.

Christine Berkesch